AoPS Wiki talk:Problem of the Day/September 17, 2011

We know that \[(999+1)^3=999^3+3(999)^2+3(999)+1\] by the Binomial Theorem. Since our given expression is one less than that, we compute \[(999+1)^3-1=1,000,000,000-1=\boxed{999,999,999}\]